Distance to a closed set is continuous.Metric space and subsetsReverse Triangle Inequality ProofEquivalence relation on a continuumHow to prove easily that a generalized ellipse is $C^1$How to prove any closed set in $mathbbR$ is $G_delta$Definitions of supremumConvergence of a point and that of its projectionCompactess of the sum closed balls whose centers is a compact set.Distance to a closed setTo show that the function $f(x) = inf d(x,x_n) : n in Bbb N $ is uniformly continuous on X.Proving that if $f:D to mathbbR$ is continuous and compact, then $f(x_n)$ is a Cauchy sequence.Prove that $ f$ is uniformly continuousShow that if $f$ continuous then $f(overlineX)subset overlinef(X)$ for all $Xsubset R$A closed set in vector space of limited sequences $l^infty$Let $f$ be a real uniformly continuous function on the bounded set $E$ in $mathbbR^1$. Prove that $f$ is bounded on $E$Let $f: mathscrB(mathbbN;mathbbR)rightarrow mathbbR$ given by $f(x) = liminf x_n.$ Prove that $f$ is continuous.Proof that the generalized inverse of an increasing right-continuous function is also right-continuousIs $A=y in BbbR: y=lim_nto inftyf(x_n), textfor some sequence~ x_n to +infty$ compact for a continuous $f$?

In Japanese, what’s the difference between “Tonari ni” (となりに) and “Tsugi” (つぎ)? When would you use one over the other?

What typically incentivizes a professor to change jobs to a lower ranking university?

Why "Having chlorophyll without photosynthesis is actually very dangerous" and "like living with a bomb"?

What are the differences between the usage of 'it' and 'they'?

Font hinting is lost in Chrome-like browsers (for some languages )

Modeling an IPv4 Address

Why don't electron-positron collisions release infinite energy?

I’m planning on buying a laser printer but concerned about the life cycle of toner in the machine

Prove that NP is closed under karp reduction?

What is the word for reserving something for yourself before others do?

Why, historically, did Gödel think CH was false?

Why are 150k or 200k jobs considered good when there are 300k+ births a month?

Has the BBC provided arguments for saying Brexit being cancelled is unlikely?

How do we improve the relationship with a client software team that performs poorly and is becoming less collaborative?

How does one intimidate enemies without having the capacity for violence?

What does "Puller Prush Person" mean?

What's the point of deactivating Num Lock on login screens?

To string or not to string

Can I make popcorn with any corn?

How to say job offer in Mandarin/Cantonese?

How does strength of boric acid solution increase in presence of salicylic acid?

Is this a crack on the carbon frame?

Is it important to consider tone, melody, and musical form while writing a song?

Can a Warlock become Neutral Good?



Distance to a closed set is continuous.


Metric space and subsetsReverse Triangle Inequality ProofEquivalence relation on a continuumHow to prove easily that a generalized ellipse is $C^1$How to prove any closed set in $mathbbR$ is $G_delta$Definitions of supremumConvergence of a point and that of its projectionCompactess of the sum closed balls whose centers is a compact set.Distance to a closed setTo show that the function $f(x) = inf d(x,x_n) : n in Bbb N $ is uniformly continuous on X.Proving that if $f:D to mathbbR$ is continuous and compact, then $f(x_n)$ is a Cauchy sequence.Prove that $ f$ is uniformly continuousShow that if $f$ continuous then $f(overlineX)subset overlinef(X)$ for all $Xsubset R$A closed set in vector space of limited sequences $l^infty$Let $f$ be a real uniformly continuous function on the bounded set $E$ in $mathbbR^1$. Prove that $f$ is bounded on $E$Let $f: mathscrB(mathbbN;mathbbR)rightarrow mathbbR$ given by $f(x) = liminf x_n.$ Prove that $f$ is continuous.Proof that the generalized inverse of an increasing right-continuous function is also right-continuousIs $A=y in BbbR: y=lim_nto inftyf(x_n), textfor some sequence~ x_n to +infty$ compact for a continuous $f$?













5












$begingroup$


I want to prove that given a metric space $(M,d)$ and $F subset M$, then the function $f_F: M to Bbb R$ given by $f_F(x) = d(x,F) = infd(x,y) : y in M$ is continuous.



Take $x in M$. If $x in F$ it is obvious. I am convinced that it is continuous, since if you take a sequence $(x_n)_n in Bbb N$ which converges to $x in M$, then the distances from the $x_n$ to $F$ go close to the distance from $x$ to $F$, since the $x_n$ go close to $x$.



But I'm having much trouble writing it. My teacher said to "just forget about the $inf$ and work with sequences", but that doesn't help me in nothing at all.



Given $epsilon > 0$ and a sequence $(x_n)_x in Bbb N$ such that $x_n to x$, I must prove that $f_F(x_n) to f_F(x)$, that is, find $n_epsilon in Bbb N$ such that: $$n > n_epsilon implies |d(x_n,F) - d(x,F)| < epsilon $$



Surely if $y in F$, we have $d(x,y) geq d(x,F)$. Other than this, I don't know how to deal with these infs, and this absolute value. I suppose I'll have to use the triangle inequality. Can someone help me do it?










share|cite|improve this question









$endgroup$
















    5












    $begingroup$


    I want to prove that given a metric space $(M,d)$ and $F subset M$, then the function $f_F: M to Bbb R$ given by $f_F(x) = d(x,F) = infd(x,y) : y in M$ is continuous.



    Take $x in M$. If $x in F$ it is obvious. I am convinced that it is continuous, since if you take a sequence $(x_n)_n in Bbb N$ which converges to $x in M$, then the distances from the $x_n$ to $F$ go close to the distance from $x$ to $F$, since the $x_n$ go close to $x$.



    But I'm having much trouble writing it. My teacher said to "just forget about the $inf$ and work with sequences", but that doesn't help me in nothing at all.



    Given $epsilon > 0$ and a sequence $(x_n)_x in Bbb N$ such that $x_n to x$, I must prove that $f_F(x_n) to f_F(x)$, that is, find $n_epsilon in Bbb N$ such that: $$n > n_epsilon implies |d(x_n,F) - d(x,F)| < epsilon $$



    Surely if $y in F$, we have $d(x,y) geq d(x,F)$. Other than this, I don't know how to deal with these infs, and this absolute value. I suppose I'll have to use the triangle inequality. Can someone help me do it?










    share|cite|improve this question









    $endgroup$














      5












      5








      5


      5



      $begingroup$


      I want to prove that given a metric space $(M,d)$ and $F subset M$, then the function $f_F: M to Bbb R$ given by $f_F(x) = d(x,F) = infd(x,y) : y in M$ is continuous.



      Take $x in M$. If $x in F$ it is obvious. I am convinced that it is continuous, since if you take a sequence $(x_n)_n in Bbb N$ which converges to $x in M$, then the distances from the $x_n$ to $F$ go close to the distance from $x$ to $F$, since the $x_n$ go close to $x$.



      But I'm having much trouble writing it. My teacher said to "just forget about the $inf$ and work with sequences", but that doesn't help me in nothing at all.



      Given $epsilon > 0$ and a sequence $(x_n)_x in Bbb N$ such that $x_n to x$, I must prove that $f_F(x_n) to f_F(x)$, that is, find $n_epsilon in Bbb N$ such that: $$n > n_epsilon implies |d(x_n,F) - d(x,F)| < epsilon $$



      Surely if $y in F$, we have $d(x,y) geq d(x,F)$. Other than this, I don't know how to deal with these infs, and this absolute value. I suppose I'll have to use the triangle inequality. Can someone help me do it?










      share|cite|improve this question









      $endgroup$




      I want to prove that given a metric space $(M,d)$ and $F subset M$, then the function $f_F: M to Bbb R$ given by $f_F(x) = d(x,F) = infd(x,y) : y in M$ is continuous.



      Take $x in M$. If $x in F$ it is obvious. I am convinced that it is continuous, since if you take a sequence $(x_n)_n in Bbb N$ which converges to $x in M$, then the distances from the $x_n$ to $F$ go close to the distance from $x$ to $F$, since the $x_n$ go close to $x$.



      But I'm having much trouble writing it. My teacher said to "just forget about the $inf$ and work with sequences", but that doesn't help me in nothing at all.



      Given $epsilon > 0$ and a sequence $(x_n)_x in Bbb N$ such that $x_n to x$, I must prove that $f_F(x_n) to f_F(x)$, that is, find $n_epsilon in Bbb N$ such that: $$n > n_epsilon implies |d(x_n,F) - d(x,F)| < epsilon $$



      Surely if $y in F$, we have $d(x,y) geq d(x,F)$. Other than this, I don't know how to deal with these infs, and this absolute value. I suppose I'll have to use the triangle inequality. Can someone help me do it?







      real-analysis metric-spaces






      share|cite|improve this question













      share|cite|improve this question











      share|cite|improve this question




      share|cite|improve this question










      asked Sep 24 '14 at 18:15









      Ivo TerekIvo Terek

      46.7k954146




      46.7k954146




















          2 Answers
          2






          active

          oldest

          votes


















          15












          $begingroup$

          From $d(x',y)leq d(x',x)+d(x,y)$, by taking infimum we get
          $d(x',F)leq d(x',x)+d(x,F)$.
          Similarly, $d(x,F)leq d(x',x)+d(x',F)$.
          Consequently, $|d(x',F)-d(x,F)|<d(x',x)$.






          share|cite|improve this answer









          $endgroup$




















            0












            $begingroup$

            Here is a more worked example without "taking infimum" using the inverse triangle inequality:



            We wish to show for all $x, y in M$, $lvert f_F(x) - f_X(y)rvert le d(x, y)$.



            Case: $d(x, z) ge d(y, z)$.



            Fix the $z$ such that both bounds are true. By the definitions of infimum, we have two bounds (let $epsilon > 0$):



            beginalign
            forall z in F quad &f_F(x) le d(x, z) \
            exists z in F quad &f_F(x) + epsilon > d(x, z)
            endalign



            By the inverse triangle inequality,



            beginalign
            & d(x, z) - d(y,z) le d(x,y) \
            implies & f_F(x) - d(y, z) le d(x,y) \
            implies & f_F(x) - f_F(y) - epsilon le d(x,y) \
            implies & f_F(x) - f_F(y) le d(x,y) + epsilon
            endalign



            Since this holds for all $epsilon > 0$, $f_F(x) - f_F(y) le d(x,y)$. The case of $d(x,z) < d(y,z)$ is similar, so we have proved $lvert f_F(x) - f_F(y)rvert le d(x, y)$. Now simply apply our definition of continuity using $delta = epsilon$ (actually $f_F$ is Lipschitz continuous with constant $1$).






            share|cite|improve this answer











            $endgroup$













              Your Answer





              StackExchange.ifUsing("editor", function ()
              return StackExchange.using("mathjaxEditing", function ()
              StackExchange.MarkdownEditor.creationCallbacks.add(function (editor, postfix)
              StackExchange.mathjaxEditing.prepareWmdForMathJax(editor, postfix, [["$", "$"], ["\\(","\\)"]]);
              );
              );
              , "mathjax-editing");

              StackExchange.ready(function()
              var channelOptions =
              tags: "".split(" "),
              id: "69"
              ;
              initTagRenderer("".split(" "), "".split(" "), channelOptions);

              StackExchange.using("externalEditor", function()
              // Have to fire editor after snippets, if snippets enabled
              if (StackExchange.settings.snippets.snippetsEnabled)
              StackExchange.using("snippets", function()
              createEditor();
              );

              else
              createEditor();

              );

              function createEditor()
              StackExchange.prepareEditor(
              heartbeatType: 'answer',
              autoActivateHeartbeat: false,
              convertImagesToLinks: true,
              noModals: true,
              showLowRepImageUploadWarning: true,
              reputationToPostImages: 10,
              bindNavPrevention: true,
              postfix: "",
              imageUploader:
              brandingHtml: "Powered by u003ca class="icon-imgur-white" href="https://imgur.com/"u003eu003c/au003e",
              contentPolicyHtml: "User contributions licensed under u003ca href="https://creativecommons.org/licenses/by-sa/3.0/"u003ecc by-sa 3.0 with attribution requiredu003c/au003e u003ca href="https://stackoverflow.com/legal/content-policy"u003e(content policy)u003c/au003e",
              allowUrls: true
              ,
              noCode: true, onDemand: true,
              discardSelector: ".discard-answer"
              ,immediatelyShowMarkdownHelp:true
              );



              );













              draft saved

              draft discarded


















              StackExchange.ready(
              function ()
              StackExchange.openid.initPostLogin('.new-post-login', 'https%3a%2f%2fmath.stackexchange.com%2fquestions%2f944659%2fdistance-to-a-closed-set-is-continuous%23new-answer', 'question_page');

              );

              Post as a guest















              Required, but never shown

























              2 Answers
              2






              active

              oldest

              votes








              2 Answers
              2






              active

              oldest

              votes









              active

              oldest

              votes






              active

              oldest

              votes









              15












              $begingroup$

              From $d(x',y)leq d(x',x)+d(x,y)$, by taking infimum we get
              $d(x',F)leq d(x',x)+d(x,F)$.
              Similarly, $d(x,F)leq d(x',x)+d(x',F)$.
              Consequently, $|d(x',F)-d(x,F)|<d(x',x)$.






              share|cite|improve this answer









              $endgroup$

















                15












                $begingroup$

                From $d(x',y)leq d(x',x)+d(x,y)$, by taking infimum we get
                $d(x',F)leq d(x',x)+d(x,F)$.
                Similarly, $d(x,F)leq d(x',x)+d(x',F)$.
                Consequently, $|d(x',F)-d(x,F)|<d(x',x)$.






                share|cite|improve this answer









                $endgroup$















                  15












                  15








                  15





                  $begingroup$

                  From $d(x',y)leq d(x',x)+d(x,y)$, by taking infimum we get
                  $d(x',F)leq d(x',x)+d(x,F)$.
                  Similarly, $d(x,F)leq d(x',x)+d(x',F)$.
                  Consequently, $|d(x',F)-d(x,F)|<d(x',x)$.






                  share|cite|improve this answer









                  $endgroup$



                  From $d(x',y)leq d(x',x)+d(x,y)$, by taking infimum we get
                  $d(x',F)leq d(x',x)+d(x,F)$.
                  Similarly, $d(x,F)leq d(x',x)+d(x',F)$.
                  Consequently, $|d(x',F)-d(x,F)|<d(x',x)$.







                  share|cite|improve this answer












                  share|cite|improve this answer



                  share|cite|improve this answer










                  answered Sep 24 '14 at 22:18









                  Fabio LucchiniFabio Lucchini

                  9,50111426




                  9,50111426





















                      0












                      $begingroup$

                      Here is a more worked example without "taking infimum" using the inverse triangle inequality:



                      We wish to show for all $x, y in M$, $lvert f_F(x) - f_X(y)rvert le d(x, y)$.



                      Case: $d(x, z) ge d(y, z)$.



                      Fix the $z$ such that both bounds are true. By the definitions of infimum, we have two bounds (let $epsilon > 0$):



                      beginalign
                      forall z in F quad &f_F(x) le d(x, z) \
                      exists z in F quad &f_F(x) + epsilon > d(x, z)
                      endalign



                      By the inverse triangle inequality,



                      beginalign
                      & d(x, z) - d(y,z) le d(x,y) \
                      implies & f_F(x) - d(y, z) le d(x,y) \
                      implies & f_F(x) - f_F(y) - epsilon le d(x,y) \
                      implies & f_F(x) - f_F(y) le d(x,y) + epsilon
                      endalign



                      Since this holds for all $epsilon > 0$, $f_F(x) - f_F(y) le d(x,y)$. The case of $d(x,z) < d(y,z)$ is similar, so we have proved $lvert f_F(x) - f_F(y)rvert le d(x, y)$. Now simply apply our definition of continuity using $delta = epsilon$ (actually $f_F$ is Lipschitz continuous with constant $1$).






                      share|cite|improve this answer











                      $endgroup$

















                        0












                        $begingroup$

                        Here is a more worked example without "taking infimum" using the inverse triangle inequality:



                        We wish to show for all $x, y in M$, $lvert f_F(x) - f_X(y)rvert le d(x, y)$.



                        Case: $d(x, z) ge d(y, z)$.



                        Fix the $z$ such that both bounds are true. By the definitions of infimum, we have two bounds (let $epsilon > 0$):



                        beginalign
                        forall z in F quad &f_F(x) le d(x, z) \
                        exists z in F quad &f_F(x) + epsilon > d(x, z)
                        endalign



                        By the inverse triangle inequality,



                        beginalign
                        & d(x, z) - d(y,z) le d(x,y) \
                        implies & f_F(x) - d(y, z) le d(x,y) \
                        implies & f_F(x) - f_F(y) - epsilon le d(x,y) \
                        implies & f_F(x) - f_F(y) le d(x,y) + epsilon
                        endalign



                        Since this holds for all $epsilon > 0$, $f_F(x) - f_F(y) le d(x,y)$. The case of $d(x,z) < d(y,z)$ is similar, so we have proved $lvert f_F(x) - f_F(y)rvert le d(x, y)$. Now simply apply our definition of continuity using $delta = epsilon$ (actually $f_F$ is Lipschitz continuous with constant $1$).






                        share|cite|improve this answer











                        $endgroup$















                          0












                          0








                          0





                          $begingroup$

                          Here is a more worked example without "taking infimum" using the inverse triangle inequality:



                          We wish to show for all $x, y in M$, $lvert f_F(x) - f_X(y)rvert le d(x, y)$.



                          Case: $d(x, z) ge d(y, z)$.



                          Fix the $z$ such that both bounds are true. By the definitions of infimum, we have two bounds (let $epsilon > 0$):



                          beginalign
                          forall z in F quad &f_F(x) le d(x, z) \
                          exists z in F quad &f_F(x) + epsilon > d(x, z)
                          endalign



                          By the inverse triangle inequality,



                          beginalign
                          & d(x, z) - d(y,z) le d(x,y) \
                          implies & f_F(x) - d(y, z) le d(x,y) \
                          implies & f_F(x) - f_F(y) - epsilon le d(x,y) \
                          implies & f_F(x) - f_F(y) le d(x,y) + epsilon
                          endalign



                          Since this holds for all $epsilon > 0$, $f_F(x) - f_F(y) le d(x,y)$. The case of $d(x,z) < d(y,z)$ is similar, so we have proved $lvert f_F(x) - f_F(y)rvert le d(x, y)$. Now simply apply our definition of continuity using $delta = epsilon$ (actually $f_F$ is Lipschitz continuous with constant $1$).






                          share|cite|improve this answer











                          $endgroup$



                          Here is a more worked example without "taking infimum" using the inverse triangle inequality:



                          We wish to show for all $x, y in M$, $lvert f_F(x) - f_X(y)rvert le d(x, y)$.



                          Case: $d(x, z) ge d(y, z)$.



                          Fix the $z$ such that both bounds are true. By the definitions of infimum, we have two bounds (let $epsilon > 0$):



                          beginalign
                          forall z in F quad &f_F(x) le d(x, z) \
                          exists z in F quad &f_F(x) + epsilon > d(x, z)
                          endalign



                          By the inverse triangle inequality,



                          beginalign
                          & d(x, z) - d(y,z) le d(x,y) \
                          implies & f_F(x) - d(y, z) le d(x,y) \
                          implies & f_F(x) - f_F(y) - epsilon le d(x,y) \
                          implies & f_F(x) - f_F(y) le d(x,y) + epsilon
                          endalign



                          Since this holds for all $epsilon > 0$, $f_F(x) - f_F(y) le d(x,y)$. The case of $d(x,z) < d(y,z)$ is similar, so we have proved $lvert f_F(x) - f_F(y)rvert le d(x, y)$. Now simply apply our definition of continuity using $delta = epsilon$ (actually $f_F$ is Lipschitz continuous with constant $1$).







                          share|cite|improve this answer














                          share|cite|improve this answer



                          share|cite|improve this answer








                          edited Mar 22 at 6:21

























                          answered Mar 22 at 0:38









                          qwrqwr

                          6,68242755




                          6,68242755



























                              draft saved

                              draft discarded
















































                              Thanks for contributing an answer to Mathematics Stack Exchange!


                              • Please be sure to answer the question. Provide details and share your research!

                              But avoid


                              • Asking for help, clarification, or responding to other answers.

                              • Making statements based on opinion; back them up with references or personal experience.

                              Use MathJax to format equations. MathJax reference.


                              To learn more, see our tips on writing great answers.




                              draft saved


                              draft discarded














                              StackExchange.ready(
                              function ()
                              StackExchange.openid.initPostLogin('.new-post-login', 'https%3a%2f%2fmath.stackexchange.com%2fquestions%2f944659%2fdistance-to-a-closed-set-is-continuous%23new-answer', 'question_page');

                              );

                              Post as a guest















                              Required, but never shown





















































                              Required, but never shown














                              Required, but never shown












                              Required, but never shown







                              Required, but never shown

































                              Required, but never shown














                              Required, but never shown












                              Required, but never shown







                              Required, but never shown







                              Popular posts from this blog

                              Lowndes Grove History Architecture References Navigation menu32°48′6″N 79°57′58″W / 32.80167°N 79.96611°W / 32.80167; -79.9661132°48′6″N 79°57′58″W / 32.80167°N 79.96611°W / 32.80167; -79.9661178002500"National Register Information System"Historic houses of South Carolina"Lowndes Grove""+32° 48' 6.00", −79° 57' 58.00""Lowndes Grove, Charleston County (260 St. Margaret St., Charleston)""Lowndes Grove"The Charleston ExpositionIt Happened in South Carolina"Lowndes Grove (House), Saint Margaret Street & Sixth Avenue, Charleston, Charleston County, SC(Photographs)"Plantations of the Carolina Low Countrye

                              random experiment with two different functions on unit interval Announcing the arrival of Valued Associate #679: Cesar Manara Planned maintenance scheduled April 23, 2019 at 00:00UTC (8:00pm US/Eastern)Random variable and probability space notionsRandom Walk with EdgesFinding functions where the increase over a random interval is Poisson distributedNumber of days until dayCan an observed event in fact be of zero probability?Unit random processmodels of coins and uniform distributionHow to get the number of successes given $n$ trials , probability $P$ and a random variable $X$Absorbing Markov chain in a computer. Is “almost every” turned into always convergence in computer executions?Stopped random walk is not uniformly integrable

                              How should I support this large drywall patch? Planned maintenance scheduled April 23, 2019 at 00:00UTC (8:00pm US/Eastern) Announcing the arrival of Valued Associate #679: Cesar Manara Unicorn Meta Zoo #1: Why another podcast?How do I cover large gaps in drywall?How do I keep drywall around a patch from crumbling?Can I glue a second layer of drywall?How to patch long strip on drywall?Large drywall patch: how to avoid bulging seams?Drywall Mesh Patch vs. Bulge? To remove or not to remove?How to fix this drywall job?Prep drywall before backsplashWhat's the best way to fix this horrible drywall patch job?Drywall patching using 3M Patch Plus Primer